Single Best Answer Flashcards

1
Q
HB, a 76-year old, 40 kg patient wishes to purchase the following herbal products. Which would you NOT sell based on the following patient record information? Current medications: Warfarin (Coumadin) 2.5mg daily x 2 years. Donepezil (Aricept) 5 mg daily x 2 months
 I-Ginkgo biloba 
II-Evening primrose oil 
III-Vitamin B complex
a. I only
b.III only side effects
c.II and III
d.All of these options
e.I and II
A

A gingko biloba risk of bleeding

How well did you know this?
1
Not at all
2
3
4
5
Perfectly
2
Q

A child has ingested an unknown substance and has evidence of respiratory depression. This symptom is usually found with poisoning due to:

a. Amphetamines
b. Atropine
c. Mushrooms
d. Kerosene
e. Opioids

A

The correct answer is e.The main toxic side effect of opioids is respiratory depression.

How well did you know this?
1
Not at all
2
3
4
5
Perfectly
3
Q

Joan Linscombe approaches you confidentially stating that she has a problem, every month for the last 5 months and has tried Miconazole and Clotrimazole ovules, 3 day therapy as per her prescribers guidance. Both were effective, but the problem recurs frequently. This time her symptoms include frequent urination and thirst. She had her menstrual period ten days ago, but it is unpredictable. She has not been to her doctor for almost a year. Based on this information, you would be MOST concerned that she could be which of the following options?

a. Using the incorrect product. Recurrent infections respond better to 6-day therapy
b. Overusing non-prescription antifungals and the subsequent bacterial overgrowth is contributing to her recurrent bacterial infections
c. Diabetic and the sugar spilling into her urine may be causing the recurrent yeast infections
d. Undergoing pre-menopausal symptoms and the fluctuation in vaginal pH is contributing to her recurrent yeast infections
e. Diabetic, but the yeast infections could not be associated with the sugar in her urine

A

The correct answer is c. The polydipsia and polyuria are the most worrying symptoms here, diabetics are more prone to all types of infection including those of the urinary tract. She is not overusing the antifungals as she is following her prescribers guidance which seems not to be excessive. Dependent on the services available at the pharmacy a dipstick urinalysis for pathogenic or hyperglycaemic markers could be performed, which may aid in further diagnosis by a suitably qualified healthcare professional.

How well did you know this?
1
Not at all
2
3
4
5
Perfectly
4
Q

A patient complains of headache and his blood pressure is 120/80. His medications include hydrochlorothiazide and hydralazine. Which of the following would seem most likely?

a. The headaches are secondary to thiazide-induced hypokalemia
b. The headaches are secondary to hydralazine therapy
c. The headaches are probably unrelated to drug therapy
d. The headaches are related to hydralazine-induced agranulocytosis
e. The headaches are caused by an temporary increased blood pressure

A

B headaches are secondary to hydralazine therapy

Hydralazine = vasodilator works by relaxing blood vessels. It produces a fall in peripheral resistance and a decrease in arterial BP, effects which induce reflex sympathetic cardiovascular responses. The concomitant use of a beta-blocker will reduce these reflex effects and enhance the anti-hypertensive effect. The use of hydralazine can result in sodium and fluid retention, producing oedema and reduced urinary volume. These effects can be prevented by concomitant administration of a diuretic.

Se: tachycardia, palpitations, headache,

How well did you know this?
1
Not at all
2
3
4
5
Perfectly
5
Q

Which of the following statements is FALSE about blood drug monitoring?

a. Carbamazepine often requires monitoring
b. Sodium valproate often requires monitoring
c. Digoxin often requires monitoring
d. Drugs which have significant pharmacokinetic variability often need monitoring
e. Monitoring should be performed immediately after the first dose

A

The correct answer is e.The main drugs which require therapeutic monitoring include those with:

  • narrow target range
    significant pharmacokinetic variability
  • a reasonable relationship between plasma concentrations and clinical effects
  • established target concentration range
  • availability of cost-effective drug assay.
How well did you know this?
1
Not at all
2
3
4
5
Perfectly
6
Q

MD, a 17 year old, presents with a purpuric skin rash. She wonders if she is having another flare-up of her eczema, but this rash looks different to previous presentations. Her patient medical record reveals the following: Allergies: penicillin History: eczema x 2.5 years epilepsy x 1 month Current medications: Betamethasone Cr 0.05% bid prn x 2.5 years Ethinyl estradiol/ levonorgestrel x 10 months Phenytoin 200mg qhs x 1 month. What is the most probable cause of her skin rash?

a. An acute flare-up of her eczema
b. A reaction to phenytoin
c. A reaction to oral contraceptives
d. An interaction between ethinyl estradiol with levonorgestrel and phenytoin
e. A reaction to the steroid

A

The correct answer is b.Phenytoin is a highly effective and widely prescribed anticonvulsant agent. Phenytoin is however, associated with both dose related side effects and hypersensitivity reactions. 5-10% of patients using phenytoin have a skin reaction.

How well did you know this?
1
Not at all
2
3
4
5
Perfectly
7
Q

The best definition for “Phlebotomy” is which of the following?

a. The act or practice of opening a vein for letting or drawing blood as a therapeutic or diagnostic measure
b. The act or practice of opening a artery for letting or drawing blood as a therapeutic or diagnostic measure
c. The branch of medicine and biology concerned with immunity
d. The branch of medicine and biology concerned with arthritic conditions
e. The branch of medicine and biology concerned with respiratory secretions

A

The correct answer is a. 1. The removal of blood from a vein, usually with a needle and syringe or other container, for diagnostic or therapeutic purposes, as in the treatment of hemochomatosis. 2. The removal of blood from a vein with a cutting instrument, formerly done to reduce blood volume as a treatment of disease. In both senses also called venesection.

How well did you know this?
1
Not at all
2
3
4
5
Perfectly
8
Q

Which of the following is a cause of hyperkalemia?

a. Acidosis
b. Crush injury
c. ACE inhibitor
d. Hypoaldosteronism
e. All of these options

A

The correct answer is e.Medication that interferes with urinary excretion:
ACE inhibitors and angiotensin receptor blockers
Potassium-sparing diuretics (e.g. amiloride and spironolactone)
NSAIDs such as ibuprofen, naproxen, or celecoxib
The calcineurin inhibitor immunosuppressants ciclosporin and tacrolimus
The antibiotic trimethoprim
The antiparasitic drug pentamidine
Mineralocorticoid deficiency or resistance, such as: Addison’s disease Aldosterone deficiency
Some forms of congenital adrenal hyperplasia Type IV renal tubular acidosis (resistance of renal tubules to aldosterone)

How well did you know this?
1
Not at all
2
3
4
5
Perfectly
9
Q

Regarding cholesterol therapy, the goal is to achieve which of the following?

a. Reduce LDL and raise triglycerides
b. Reduce LDL and raise HDL
c. Raise LDL and raise HDL
d. Raise LDL and reduce triglycerides
e. None of these options

A

The correct answer is b.Total cholesterol LDL (low-density lipoprotein cholesterol, also called “bad” cholesterol) HDL (high-density lipoprotein cholesterol, also called “good” cholesterol)Triglycerides (fats carried in the blood from the food we eat. Excess calories, alcohol, or sugar in the body are converted into triglycerides and stored in fat cells throughout the body.)

How well did you know this?
1
Not at all
2
3
4
5
Perfectly
10
Q

Mr. H is seeking advice about this wife, Mrs. H (aged 30). They have just returned from their holiday in Australia and Mrs. H is complaining of a stiff inflamed lower leg. He says it is quite red and hot. She is not taking any medication except her usual combined oral contraceptive pill, What is the most appropriate advice?

a. Seek medical help immediately
b. Sell ibuprofen 400mg and tell her to take it regularly for five days
c. Recommend that she follows the RICE approach and seek further advice if no improvement after seven days
d. Sell aspirin for DVT prophylaxis
e. Make an appointment to see her GP

A

The correct answer is a. The patient has been on a long haul flight (above 6 hours). The symptoms he is describing could be a potential DVT which requires urgent medical referral. Management of this condition if often sough from a hospital where low molecular weight heparin (LMWH) is usually given. It is an important part of a pharmacists daily job to look out for these types of “red flag” warning signs that a patients life may be in danger and then be able to sign post correctly.

How well did you know this?
1
Not at all
2
3
4
5
Perfectly
11
Q

Which of the following tests will give a decreased test result in patient suffering from renal failure?

a. Serum Creatinine
b. Blood Urea Nitrogen
c. Creatinin clearance
d. Urea breath Test
e. All of these options

A

C

How well did you know this?
1
Not at all
2
3
4
5
Perfectly
12
Q

Which of the following medicines will increase Theophylline serum levels if combined with Theophylline?

a. Carbamazepine
b. Tobacco smoking
c. Phenytoin
d. Ciprofloxacin HCl
e. All of these options

A

The correct answer is d.

How well did you know this?
1
Not at all
2
3
4
5
Perfectly
13
Q

Patient name: DA Age: 60 years old Gender: male Allergies: No known allergies Medical conditions: Type 2 Diabetes, Hypercholesterolemia, Occasional angina Other: Half-marathon walker, eats grapefruit Medications:
Atorvastatin (Lipitor) 20 mg qhs - start 2 yrs ago
Metformin (Glucophage) 500 mg TID - start 2 yrs ago
Sildenafil (Viagra) 100 mg hs prn - start 2 months ago Nitroglycerin (Nitrolingual) 0.4 mg Spray 1-2 sprays sl prn -initiate-start 3 weeks ago

DA requests a repeat of his sildenafil (Viagra) prescription.

You would be concerned about all of the following EXCEPT:

a. Atorvastatin with grapefruit
b. Sildenafil with nitroglycerin
c. Metformin with atorvastatin
d. Heart conditions and marathons
e. Sildenafil with grapefruit

A

correct answer is c.As the patient has angina the marathon running may be an issue, the others and standard interactions.

How well did you know this?
1
Not at all
2
3
4
5
Perfectly
14
Q

Which one of the following is least likely to cause a significantly elevated level of ALT (SGPT)?

a. Viral hepatitis
b. Diabetes
c. Congestive heart failure
d. Liver damage
e. Transient ischaemic event

A

The correct answer is e.
Significantly elevated levels of ALT (SGPT) often suggest the existence of other medical problems such as viral hepatitis, diabetes, congestive heart failure, liver damage, bile duct problems, infectious mononucleosis, or myopathy, so ALT is commonly used as a way of screening for liver problems. Elevated ALT may also be caused by dietary choline deficiency. However, elevated levels of ALT do not automatically mean that medical problems exist. Fluctuation of ALT levels is normal over the course of the day, and they can also increase in response to strenuous physical exercise.

How well did you know this?
1
Not at all
2
3
4
5
Perfectly
15
Q
Patient Name: FJ
 Age: 40 years old
 Allergies: No known allergies
 History: Smoker
 Current medications: Zopiclone 7.5 mg qhs prn x 30

FJ presents the following new prescription:
Clarithromycin 500mg bid x 7 days
Metronidazole 500 mg bid x 7 days
Bismuth subsalicylate (Pepto Bismol) ii tabs qid x 7 days

What is the most likely diagnosis according to this new prescription drug regimen?

a. Helicobacter pylori infection induced peptic ulcer
b. Salmonella intestinal infection
c. Intestinal amoebiasis
d. Escherichia coli intestinal infection
e. Gardnerella vaginitis

A

The correct answer is a
ALT is commonly measured clinically as a part of a diagnostic evaluation of hepatocellular injury, to determine liver health. When used in diagnostics, it is almost always measured in international units/liter (IU/L). Significantly elevated levels of ALT (SGPT) often suggest the existence of other medical problems such as viral hepatitis, diabetes, congestive heart failure, liver damage, bile duct problems, infectious mononucleosis, or myopathy, so ALT is commonly used as a way of screening for liver problems.

How well did you know this?
1
Not at all
2
3
4
5
Perfectly
16
Q

Which of the following is an example of an enzyme inhibitor?

a. Grisoefulvin
b. Phenytoin
c. Phenobarbitone
d. Fluconazole
e. Smoking cigarettes

A

The correct answer is d.

How well did you know this?
1
Not at all
2
3
4
5
Perfectly
17
Q

Patient Name: HR, Age: 79 yo, Allergies: Allopurinol
Current medications:
Zopiclone 7.5 mg hs prn
Hydrochlorothiazide 25 mg daily
Potassium chloride 600 mg (slow K) ii bid
Digoxin 0.25 mg daily

HR has a prescription for: Verapamil SR 240 mg daily

Upon reviewing the patient record, you would:

a. Call the doctor regarding a possible hydrochlorothiazide interaction
b. Call the doctor to decrease the dose of verapamil
c. Dispense as written; warn the patient to notify you of any new gastrointestinal symptoms
d. Call the doctor regarding a potential digoxin interaction
e. Call the doctor regarding a potential verapamil hypersensitivity

A

The correct answer is e. Serum digoxin concentration rise by 60-75% due to decreased renal tubular secretion and nonrenal clearance mechanisms. Additionally, there appears to be a synergistic effect of slowing impulse conduction and muscle contractility, leading to bradycardia and possible heart block.

How well did you know this?
1
Not at all
2
3
4
5
Perfectly
18
Q

ALT is most commonly measured clinically as a part of a diagnostic evaluation of which of the following?

a. Hepatocellular injury
b. Brain injury
c. Cardiac injury
d. Respiratory injury
e. Ocular nerve degradation

A

The correct answer is a.
ALT is commonly measured clinically as a part of a diagnostic evaluation of hepatocellular injury, to determine liver health. When used in diagnostics, it is almost always measured in international units/liter (IU/L). Significantly elevated levels of ALT (SGPT) often suggest the existence of other medical problems such as viral hepatitis, diabetes, congestive heart failure, liver damage, bile duct problems, infectious mononucleosis, or myopathy, so ALT is commonly used as a way of screening for liver problems.

How well did you know this?
1
Not at all
2
3
4
5
Perfectly
19
Q

Which of the following options would be considered the most common cause of HYPOcalcemia?

a. Rickets
b. Osteomalacia
c. Renal failure
d. Massive blood transfusion
e. Idiopathic hypoparathyroidism

A

The correct answer is c

How well did you know this?
1
Not at all
2
3
4
5
Perfectly
20
Q

Which of the following white blood cells is capable of phagocytosis?

a. Basophil
b. Eosinophil
c. Lymphocyte
d. Neutrophil
e. Platelets

A

The correct answer is d.Phagocytosis is the process by which a cell can ingest and digest other cells. The two WBCs that are phagocytic in nature are neutrophils and monocytes.

How well did you know this?
1
Not at all
2
3
4
5
Perfectly
21
Q

Torsade de pointes is a lethal complication of ventricular tachycardia.

It can be caused by drugs such as sotalol and hypokalaemia.

What is the treatment for Torsades de Pointes?

A. IV Mg Sulfate
B. IV amiodarone
C. IV sodium chloride
D. IV flumenazil

A

A iv mg sulfate

How well did you know this?
1
Not at all
2
3
4
5
Perfectly
22
Q

Mrs L comes into the pharmacy complaining of congestion and productive cough. She takes lisinopril 5mg regularly for her BP.

Which product can you sell her?

A. Robitussin chesty cough
B. Sudafed
C. Lemsip max sinus and flu
D. Day and night nurse caps

A

A correct. Guaifenesin expectorajt. If you’re taking medicines for highblood pressureyou shouldn’t takeRobitussinmucus cough and congestion relief as well, because the pseudoephedrine may make yourblood pressuremedicine less effective.
B. Sudafed has pseudoephedrine
C. Lemsipproducts contain decongestants (paracetamol and phenylephrine) which constrictbloodvessels; this may worsen raisedBP. ThereforeLemsip, which contains decongestant, should not be taken.
D. Day; paracetamol, pseudoephedrine, pholcodine
Night; paracetamol, promethazine, dextromethorphan

How well did you know this?
1
Not at all
2
3
4
5
Perfectly
23
Q

Match the antidotes to the drug

  1. Protamine sulfate
  2. Pyridoxine
  3. Physostigmine
  4. IV Flumenazil
  5. Naloxone
  6. Glucagon
  7. Activated charcoal
  8. N- acetylcysteine
  9. Phytomenadione
A. Isoniazid
B. Benzodiazepines
C. Beta blockers
D. Acetaminophen
E. Many toxins 
F. Heparin
G. Anticholinergics
H. Warfarin
I. Opioids
A
Naloxone = opioids
Physostigmine = anticholinergics 
Protamine sulfate = heparin
Pyridoxine = isoniazid
Glucagon = beta blockers
N- acetylcysteine = acetaminophen 
Phytomenadione = warfarin
Flumenazil = benzodiazepines 
Activated charcoal = many toxins
How well did you know this?
1
Not at all
2
3
4
5
Perfectly
24
Q

Match the vitamin to the following statements.
Vitamin A, B, C, D, E, K

  1. Helps with the common cold
  2. Deficiency can cause scurvy
  3. Contraindicated pregnancy
  4. Deficiency can cause rickets
  5. Tocopherol
  6. Green leafy veg
  7. Given to all newborns to prevent neonatal haemorrhage
  8. Deficiency petechiae
  9. Warfarin + vitamin => enhanced anticoagulation
  10. Antioxidant effect
  11. Deficiency beriberi
  12. Thiamine
A
  1. Helps with the common cold - C
  2. Deficiency can cause scurvy - C
  3. Contraindicated pregnancy - A
  4. Deficiency can cause rickets - D
  5. Tocopherol - E
  6. Green leafy veg - K
  7. Given to all newborns to prevent neonatal haemorrhage - K
  8. Deficiency petechiae - B3 (niacin) and C
  9. Warfarin + vitamin => enhanced anticoagulation - K
  10. Antioxidant effect - C
  11. Deficiency beriberi - B1
  12. Thiamine - B1
How well did you know this?
1
Not at all
2
3
4
5
Perfectly
25
Q
Max sumatriptan you can take in 24 hrs?
A. 4 tabs
B. 6 tabs
C. 2 tabs
D. 1 tabs
E. No restrictions on maximum dose
A

Max dose 300mg in 24 hrs
Comes in 50mg or 100mg
B

How well did you know this?
1
Not at all
2
3
4
5
Perfectly
26
Q
What dose of prednisolone would you expect for an acute asthma attack?
A. 30 - 40mg for 5 days
B. 30 - 40mg for 3 days
C. 40 - 50mg for 5 days
D. 40 - 50mg for 3 days
A

C. 40 - 50mg for at least 5 days

How well did you know this?
1
Not at all
2
3
4
5
Perfectly
27
Q

The MHRA released advice regarding Domperidone, in particularly the duration of its use. The advice restricts the dose, indication and duration of use, due to the risk of serious cardiac side-effects.
How many days should Domperidone be prescribed for?
A) Up to 3 days
B) Up to 4 days
C) Up to 5 days
D) Up to 7 days

A

D 7 days

Domperidone is no longer indicated for the relief of n+v in children < 12 yrs or those weighing < 35 kg. A European review concluded that domperidone is not as effective in this population as previously thought and alternative treatments should be considered. Healthcare professionals are to use the lowest effective dose for the shortest possible duration (max. treatment duration should not usually exceed 1 week).

How well did you know this?
1
Not at all
2
3
4
5
Perfectly
28
Q

You are currently conducting a medication history for Mr N who was admitted to the ward this morning.
You gather that he is currently on:

Tegretol® Prolonged Release 200mg: 1 BD
Sertraline 50mg: 1 OD
Ramipril 10mg: 1 OD
Paracetamol 500mg: 1-2 QDS PRN

He presented being confused, lethargic and generally fatigued. You review his blood results and find that there is an electrolyte disturbance.

Based on the medication Mr N is taking, and the clinical presentation he presented with, which electrolyte disturbance is most likely to have occured?
A) Hypokalaemia
B) Hyponatremia
C) Hypernatremia
D) Hypocalcemia
A

Carbamazepine => hyponatraemia
Sertraline => hyponatraemia
B

How well did you know this?
1
Not at all
2
3
4
5
Perfectly
29
Q

You are conducting an anti-depressant medication review with 79 yo Mr PL. You have been seeing Mr PL who was commenced on Sertraline 50mg 1 OD a year and a half ago since the passing of his wife. Today Mr PL informs you he feels as though he does not need this medication anymore and is informing you, he is in a much better place. He has, over the last week or so started to wonder if he can come off this medication.
How long should antidepressant treatment be continued in Mr PL following
remission?
A) 1 week
B) 4 weeks
C) 6 months
D) 12 months

A

C

The dose should preferably be reduced gradually over about 4 weeks, or longer if withdrawal symptoms emerge (6 months in patients who have been on long-term maintenance treatment).

How well did you know this?
1
Not at all
2
3
4
5
Perfectly
30
Q
Mr A, 47 years old, has been coming into the practice to talk about their ongoing depression. Mr A has been undergoing psychological and psychosocial interventions, however this has not seemed to help. The GP is looking at possible pharmacological treatment options. Mr A is at very low risk of self-harm and has no other medical problems. Which of the following treatments below would be most appropriate?
A) Citalopram
B) Amitriptyline
C) Clozapine
D) Phenelzine
A

A. SSRIs are better tolerated and are safer in overdose than other classes of antidepressants and should be considered first-line for treating depression.

Citalopram = SSRI
Amitriptyline = TCA
Clozapine = antipsychotic 
Phenelzine = moai
How well did you know this?
1
Not at all
2
3
4
5
Perfectly
31
Q
Miss R, has been diagnosed with epilepsy. She has come into the practice to see the GP after having a seizure yesterday. Whilst having a seizure, she injured herself resulting in being in severe pain. What would be the least appropriate analgesic to prescribe to Miss R?
A) Paracetamol
B) Tramadol
C) Codeine
D) Naproxen
A

B
Seizuresare a rare side effect of tramadol. Tramadol-relatedseizuresare short, tonic-clonic seizuresthat, like other drug- related seizures, are self-limiting. Thisoccurs at both low and high doses.

How well did you know this?
1
Not at all
2
3
4
5
Perfectly
32
Q

Mr I has presented a prescription for some Matrifen® (Fentanyl) patches. You are counselling Mr I on these patches, as this is the first time he has had them.
Which of the following statements is most appropriate regarding counselling for these patches?
A) Remove and change patch every 72 hours, place replacement patch on the same area.
B) Remove and change patch every 48 hours, place replacement patch on a different area.
C) Remove and change patch every week, place replacement patch on a different area.
D) Remove and change patch every 72 hours, place replacement patch on a different area.

A

D

How well did you know this?
1
Not at all
2
3
4
5
Perfectly
33
Q

The MHRA released advice regarding Metoclopramide, in
particularly the duration of its use. The advice reinforces the fact, that prolonged use can possibly cause serious neurological adverse
effects.
How many days should Metoclopramide be prescribed for?
A) Up to 3 days
B) Up to 4 days
C) Up to 5 days
D) Up to 7 days

A

C

In adults > 18 yrs, metoclopramide should only be used for prevention of postop, radiotherapy-induced, delayed (but not acute) chemotherapy-induced, and symptomatic treatment of n+v, including that associated with acute migraine (where it may also be used to improve absorption of oral analgesics ~gastric stasis);
Only be prescribed for short-term use (up to 5 days);
Usually 10 mg, up to tds; max. daily dose 500mcg/kg;
IV doses be given as slow bolus over at least 3 min;
Oral liquid formulations be given via an appropriately designed, graduated oral syringe to ensure dose accuracy.

This advice does not apply to unlicensed uses of metoclopramide (e.g. palliative care).

How well did you know this?
1
Not at all
2
3
4
5
Perfectly
34
Q

You are carrying out a polypharmacy medication review with Mr N.C. As part of the review you check his adherence and compliance to his medication regime.
Mr N.C informs you that on the whole he is happy with his medications, apart from one.
This particular medication which is part of Mr N.C treatment regime is causing him a dry mouth which does not like, and therefore does not take it regularly as he should.
Which of the medications below is likely to be the cause for Mr N.C dry mouth?
A) Salbutamol
B) Salmeterol
C) Beclometasone Dipropinate
D) Tiotropium

A

D
Tiotropium is an antimuscarinic bronchodilator. It works by relaxing and opening the air passages to the lungs to make breathing easier.Tiotropium acts mainly on M3 muscarinic receptors located in the airways to produce smooth muscle relaxation and bronchodilation. Most common adverse effect is dry mouth 👄

How well did you know this?
1
Not at all
2
3
4
5
Perfectly
35
Q

Mr L, 58 years old has come into the practice today to see his regular GP. Mr L has recently been experiencing pain in his back.
Below is the list of medication Mr Lis currently taking.
Priadel 400mg Tablets
Levothyroxine 100mcg Tablets
Olanzapine 10mg Tablets
What would be the least suitable analgesic to prescribe, considering Mr L’s medication?
A) Naproxen
B) Paracetamol
C) Codeine
D) Meptazinol

A

A
Taking lithium along with NSAIDs might increase the risk of lithium side effects. Avoid taking lithium supplements and NSAIDs at the same time. Some NSAIDs include ibuprofen, indomethacin, naproxen, piroxicam, aspirin, and others.

How well did you know this?
1
Not at all
2
3
4
5
Perfectly
36
Q

Miss U, 73 years old is new to the practice. You are currently conducting a meds reconciliation from her previous practice notes.
From the notes you can see that she has been stabilised on a brand of Lithium Citrate for 20 years, and her bloods from the last 3 years all show Lithium being in range.
She also has her annual secondary care mental health review with the psychiatry team.
How often should Miss U come in for monitoring for her Lithium?
A) 2 weekly
B) Monthly
C) 3 monthly
D) 6 monthly

A

D

Routine serum-lithium monitoring should be performed weekly after initiation and after each dose change until concentrations are stable, then every 3 months for the first year, and every 6 months thereafter. Patients who are 65 years and older, taking drugs that interact with lithium, at risk of impaired renal or thyroid function, raised calcium levels or other complications, have poor symptom control or poor adherence, or whose last serum-lithium concentration was 0.8 mmol/litre or higher, should be monitored every 3 months. Additional serum-lithium measurements should be made if a patient develops significant intercurrent disease or if there is a significant change in a patient’s sodium or fluid intake.

How well did you know this?
1
Not at all
2
3
4
5
Perfectly
37
Q
Which of the following treatments for gout do not induce fluid retention and can be co-administered with anticoagulants?
A. Febuxistat
B. Naproxen
C. Allopurinol
D. Canakinumab
E. Omeprazole
A

The use ofcolchicineis limited by the development of toxicity at higher doses. However unlike NSAIDs, it does not induce fluid retention; moreover, it can be co-administered with anticoagulants.

How well did you know this?
1
Not at all
2
3
4
5
Perfectly
38
Q
A pt of yours suffers from gout and standard treatment has not been effective. He has recently had his gouty arthritis attack making it the 4th one this year. What drug can be initiated at this stage?
A. methotrexate
B. benzodiazepine
C. xanthine oxidase inhibitor
D. interleukin-1 inhibitor
E. corticosteroid
A

An interleukin-1 inhibitor, such ascanakinumab, can be considered for treatment of frequent gouty arthritis attacks (at least 3 in the previous 12 months) in patients who have an inadequate response to standard treatment.

How well did you know this?
1
Not at all
2
3
4
5
Perfectly
39
Q

A pt has been started on allopurinol which has been titrated up to a higher dose every four weeks to reach <300micromol/L serum uric acid.
He has recently had an acute gout attack fir which he was given colchicine in hospital. For how long should he continue to take colchicine for?
A. 7 days
B. 30 days
C. 28 days
D. 3 months
E. 6 months

Bonus: if the patient were contraindicated to the use of colchicine what is the alternative?

A

E
The initiation or up-titration of urate-lowering therapy may precipitate an acute attack, and thereforecolchicineshould be considered as prophylaxis and continued for up to 6 months.

A low-dose NSAID with gastro-protection is an alternative in patients who have contra-indications tocolchicine. If an acute attack develops during treatment, the urate-lowering therapy should continue at the same dosage and the acute attack treated separately.

How well did you know this?
1
Not at all
2
3
4
5
Perfectly
40
Q

Pityriasis rosea is a relatively common skin condition that causes a temporary rash of raised red scaly patches on the body. What is the name of the rash that appears in the first 2-3 days?
A. Widespread rash
B. Heralds patch
C. Vitiligo

A

B

How well did you know this?
1
Not at all
2
3
4
5
Perfectly
41
Q

Mr. P attends your pharmacy based travel clinic with his 9-year old son. He asks for advice about malaria tablets for their upcoming trip to Ghana. Mr. P is currently taking citalopram 10 mg and amlodipine 10 mg. His son does
not take any regular medication. Neither Mr P nor his son have any known allergies.
You check the appropriate website and confirm that there is a high risk of malaria in Ghana and the
recommended anti-malarials are atovaquone with proguanil hydrochloride or doxycycline or mefloquine.
Considering Mr P would prefer for both him and his son to take the same anti-malarial tablets, which of the
following would be the most appropriate?
A. Atovaquone alone
B. Atovaquone combined with proguanil
C. Doxycycline
D. Mefloquine
E. Proguanil alone

A

Answer: B - Atovaquone/Proguanil

  • Doxycycline not suitable for children <12 years.
  • Mefloquine not suitable for Mr P due to history of depression.
  • Atovaquone/proguanil suitable for both
How well did you know this?
1
Not at all
2
3
4
5
Perfectly
42
Q

Miss J, aged 16-years, comes into the pharmacy asking to speak to the pharmacist privately. Miss J explains that she is experiencing very heavy periods and often bleeds through her bedding at night. She also needs to change
her sanitary protection every two hours throughout the day which is preventing her from doing her normal
everyday activities. She read online that there are medicines she can buy to help with this. She is currently not taking any other medicines and has no allergies.
What would be the most appropriate response to Miss J?
A. No action required, condition is self-limiting
B. Refer to GP
C. Supply ibuprofen tablets
D. Supply paracetamol tablets
E. Supply tranexamic acid tablets

A

B

patient has symptoms of menorrhagia, this can be treated OTC with tranexamic acid however as patient is
16 years old, we cannot supply (women aged 18-45 years), therefore must refer patient to GP to get prescription.
Paracetamol, ibuprofen would not be sufficient for treatment.

How well did you know this?
1
Not at all
2
3
4
5
Perfectly
43
Q

Mrs G, a 39-year-old woman, comes into your pharmacy asking to purchase Alli© (orlistat) to aid weight loss.
Which of the following statement is INCORRECT?
A. Alli© may potentially impair the absorption of fat-soluble vitamins
B. Alli© should be taken in conjunction with high fat, mildly hypocaloric diet
C. Alli© should only be sold to overweight adults with a BMI >28kg/m²
D. Maximum daily dose of 180mg in divided doses.
E. Treatment with OTC Alli© should not exceed 6 months

A

B

Statement should say low fat, mildly hypocaloric diet.

How well did you know this?
1
Not at all
2
3
4
5
Perfectly
44
Q

Mr O, a 47-year-old man, requests to buy OTC Pantoloc Control© (Pantoprazole)for the treatment of heartburn.
He is not taking any other medicines and has no allergies.
Which of the following signs/symptoms would make the supply unsuitable?
A. Acidic taste in mouth after meals
B. Burning sensation in middle of the chest
C. Family history of colon cancer
D. Increased appetite
E. Unintentional weight loss

A

E

How well did you know this?
1
Not at all
2
3
4
5
Perfectly
45
Q

Mr. K, a 65-year-old man, comes into the pharmacy asking to speak to the pharmacist regarding the purchase of Viagra Connect® (sildenafil 50mg tabs). You speak to Mr K in the consultation room and ask questions to determine whether you can make a supply. After questioning, you decide to refer Mr. K to his GP as you are unable to supply the medicine as a Pharmacy medicine.
Which of the following is the most likely reason that the supply for Mr. K was NOT appropriate?
A. He drinks alcohol
B. He is 65 years old
C. He is currently taking doxazosin
D. He is currently taking lansoprazole
E. He is a smoker

A

C Interaction between sildenafil and alpha blockers. Doxazosin causes significant hypotensive effects when
given with sildenafil.

How well did you know this?
1
Not at all
2
3
4
5
Perfectly
46
Q

Master M, an 8-year-old child, has been diagnosed with scarlet fever. He is known to be allergic to penicillin.
Which of the following would be the most appropriate option for Master M to treat his condition?
A. Azithromycin 300 mg once daily for 5 days
B. Doxycycline 200 mg daily for 7 days
C. No treatment, the condition is self-limiting.
D. Phenoxymethylpenicillin 250 mg qds for 10 days
E. Treatment with paracetamol alone

A

A, Azithromycin.
- not B & D, due to child’s age and history of penicillin allergy
- not C & E. Paracetamol can be given alongside the antibiotics.
Scarlet fever should be treated promptly with antibiotics to prevent complications, regardless of the severity. Azithromycin is recommended for ppl with penicillin allergy.
1st line is Phenoxymethylpenicillin qds for 10 days, however, this is unsuitable due to history of allergy.
Doxycycline is unsuitable as it is not a recommended antibiotic for this condition, plus it should not be prescribed for children <12 years.

How well did you know this?
1
Not at all
2
3
4
5
Perfectly
47
Q

Miss V, a 10-year-old child, has been experiencing fever, runny nose, and a red rash for the past 72 hours. Her mother took her to see the GP who recommended paracetamol and plenty of fluids and said the child does not need to stay off school. The GP found no other symptoms, including no signs of photophobia, stiff neck or vomiting.
Which of the following conditions is the most likely diagnosis for Miss V’s condition?
A. Chickenpox
B. Impetigo
C. Meningitis
D. Scarlet fever
E. Slapped cheek syndrome

A

E
Once diagnosed with slapped cheek syndrome, you do not need to keep your child off school because once the rash appears, they are no longer infectious. It is usually a mild, self-limiting illness.
For children and adults (not pregnant) with suspected parvovirus B19 infection, management includes
advice on:
• Symptom relief, such as fluids, analgesia, and rest.
• The need for exclusion from school, nursery, or work (if appropriate). Note: this is not usually necessary as the
person is no longer infectious one day after any rash or symptoms develop.
Scarlet fever, impetigo and meningitis would all require antibiotic treatment. If your child has chickenpox, they need to be kept off school until all the spots have crusted over.

How well did you know this?
1
Not at all
2
3
4
5
Perfectly
48
Q

Mrs G, aged 73 years, has recently been discharged from hospital following a stroke. The hospital has started Mrs G on clopidogrel, however the consultant is concerned about the risk of gastrointestinal bleeding and would like to co-prescribe gastrointestinal protection.
Which of the following medicines would be the least appropriate to co-prescribe?
A. Lansoprazole
B. Omeprazole
C. Pantoprazole
D. Rabeprazole
E. Ranitidine

A

B
Interaction between clopidogrel and proton pump inhibitors. Proton pump inhibitors are inhibitors of CYP2C19 with varying degrees of potency. Therefore,PPIsmay inhibit activation ofclopidogrelto its active metabolite via CYP2C19 leading to decreased serum concentrations of the active metabolite and potentially reducing the antiplatelet effect ofclopidogrel.
FDA, MHRA and EMA discourage use of omeprazole and esomeprazole in patients taking clopidogrel (this is also
shown in BNF appendix 1).
Pantoprazole is the least likely to interact and lansoprazole and rabeprazole are also suitable alternatives.

How well did you know this?
1
Not at all
2
3
4
5
Perfectly
49
Q

A GP practice based pharmacist is carrying out a medication review for Mrs L, aged 67 years. The pharmacist is considering the appropriateness of a statin for Mrs L, given her myocardial infarction 2 years ago. According to her current and past medication lists, Mrs L has never been prescribed a statin. You cannot see from her notes any clinical reason why a statin would not be appropriate and so decide to initiate one.
Which statin would be considered first line for Mrs L?
A. Atorvastatin 20 mg o.d.
B. Atorvastatin 80 mg o.d.
C. Rosuvastatin 20 mg o.d.
D. Simvastatin 20 mg o.d.
E. Simvastatin 80 mg o.d.

A

B: atorvastatin 80 mg.
NICE - Atorvastatin 80mg od 1st line for 2ndary prevention of CVD.
https://cks.nice.org.uk/topics/lipid-modification-cvd-prevention/management/lipid-therapy-secondary-prevention-of-cvd/

Prescribe a lower dose if:
There aredrug interactions.
There is an increased risk ofadverse events.
The person requests to start at a lower dose.

High intensity statin treatment is the most clinically effective option for the prevention of CVD. However, statins at any intensity reduce CVD risk compared with no treatment.

How well did you know this?
1
Not at all
2
3
4
5
Perfectly
50
Q

Mr H, a 45-year old patient at your community pharmacy, brings in a new prescription for linagliptin 5 mg tablets once daily. You notice that he was previously prescribed sitagliptin 100 mg tablets once daily, which he had been taking for 2 years. Mr H has type 2 diabetes mellitus which is currently well controlled. Mr H’s only other regular medication is ibuprofen 400 mg tds for back pain.
Which of the following is the most likely reason for the switch to linagliptin?
A. Linagliptin has fewer side effects than sitagliptin
B. NICE recommends linagliptin over sitagliptin to reduce the risk of hypoglycaemia
C. The patient’s HbA1C is not controlled
D. The patient’s liver function has deteriorated and so linagliptin is more appropriate
E. The patient’s renal function has deteriorated and so linagliptin is more appropriate

A

E, the patient’s renal function deteriorated.

NICE does not recommend one gliptin over another. If HbA1c not controlled, the doctor would prescribe another antidiabetic, not switch to another gliptin.
Little effect of sitagliptin and linagliptin on liver function.
For renal function, dose adjustments would be required for sitagliptin, no dose adjustments required for linagliptin in renal impairment.

NICE CKS: https://cks.nice.org.uk/topics/diabetes-type-2/prescribing-information/gliptins/

• Gliptins are inhibitors of the enzyme dipeptidyl peptidase 4 (DPP-4). DPP-4 plays a major role in glucose metabolism by rapidly degrading the incretins (glucose-dependent insulinotropic polypeptide [GIP] and glucagon-like peptide 1 [GLP-1]) which stimulate postprandial insulin secretion and suppress glucagon secretion. Inhibition of DPP-4 therefore results in increased circulating levels of GIP and GLP-1 following the ingestion of food, and to increased insulin secretion and reduced glucagon secretion.
• There are currently five gliptins available in the UK: sitagliptin, saxagliptin, vildagliptin, linagliptin, and
alogliptin.

51
Q

You receive a Rx for sodium valproate for Miss R, a 35-yo woman. You see this as a good learning opportunity to quiz your pharmacy team regarding the use of sodium valproate in women of childbearing age.
Which of the following statements regarding the use of sodium valproate is INCORRECT?
A. Pharmacists must discuss the risks with female patients each time sodium valproate is dispensed.
B. Sodium valproate can be used as a last line therapy in pregnancy to manage bipolar disorder.
C. Sodium valproate should be dispensed in whole packs where possible, with warning labels.
D. The patient’s full blood count and liver function must be monitored during treatment.
E. Women of child-bearing age taking sodium valproate must be on the pregnancy prevention programme

A

B
sodium valproate is contraindicated for management of bipolar disorder in pregnant women. Valproate is contraindicated as treatment for epilepsy during pregnancy unless there is no suitable alternative to treat epilepsy

52
Q

Miss AL comes into your pharmacy asking to speak to the pharmacist. She has been taking antibiotics for the past 3 days and has noticed that her urine has turned dark yellow, and occasionally brown.
What of the following antibiotics is most likely to have caused this?
A. Clindamycin 150 mg caps
B. Co-amoxiclav 500/125 tabs
C. Nitrofurantoin 100 mg MR Caps
D. Rifampicin 150 mg caps
E. Trimethoprim 100 mg tabs

A

C

Nitrofurantoin can colour urine dark yellow/brown, which is harmless. Rifampicin can turn urine a reddish orange colour.

53
Q

Mr N has brought in a new prescription for Priadel© 200 mg MR tabs (lithium carbonate) for the management of bipolar disorder. Mr N has been told by the prescriber that he will require frequent monitoring whilst he is taking this medicine.
Which of the following tests is least likely to be conducted whilst Mr N is taking Priadel©?
A. Body Mass Index
B. Liver function test
C. Renal function test
D. Serum electrolytes
E. Thyroid function test

A

B. LFT

BNF recommends monitoring body weight/BMI, serum electrolyte, eGFR, thyroid function every 6 months.

54
Q

Mrs C, a 33-yo woman, comes into the pharmacy asking to buy something OTC to help with nausea, vomiting and stomach-ache. Mrs C’s appetite has reduced over the past few weeks and she has been feeling more tired than usual. You notice that the whites of her eyes are slightly yellow. Upon further questioning, Mrs C tells you she is currently taking folic acid and MTX for the management of Crohn’s disease. Mrs C has not tried anything yet to relieve her symptoms.
Which of the following would be the most appropriate advice for Mrs C?
A. Advise Mrs C to try Gaviscon© Advance for 48 hours and see her GP if symptoms persist.
B. Advise Mrs C to purchase omeprazole over the counter.
C. Recommend a pregnancy test as a first priority.
D. Recommend whitening eye drops and a tonic to help boost Mrs C’s appetite.
E. Refer Mrs C to her GP urgently.

A

E
nausea, vomiting, stomach-ache, reduced appetite, fatigue, yellowing of the whites of the eye are all possible signs of liver toxicity due to methotrexate - would warrant urgent referral to GP.

Interaction between methotrexate and omeprazole - omeprazole decreases the clearance of methotrexate,
manufacturer advices use with caution or avoid.

55
Q

Miss T has been taking azathioprine for the last 4 months. She is in the pharmacy collecting her mother’s repeat medication and mentions that she has a bruise on her left leg which seems to be taking a while to go down. The bruising has also started spreading up her leg. The bruise first appeared around 3 weeks ago.
Which of the following options would be the most appropriate advice to give Miss T?
A. Advise Miss T that the symptoms she is describing are unlikely to be due to her medication.
B. Miss T should contact her GP straight away and explain that she has unexplained bruising.
C. Miss T should buy arnica cream and see if that resolves the bruising.
D. Miss T should call 999 as she requires an ambulance immediately.
E. Miss T should stop taking the azathioprine straight away and call her consultant for an appointment in the
next 2 weeks.

A

B

unexplained bruising needs to be reported to their GP immediately. Bruising could be a sign of bone marrow
suppression related to the azathioprine. Inexplicable bleeding is also a sign, as is an infection.
A. No - incorrect
C. No - this will delay medical advice and not solve the problem
D. No - this isn’t an emergency requiring an ambulance as the GP can deal with this .
E. No – this will delay medical advice and the GP can manage this

“Patients and their carers should be warned to report immediately any signs or symptoms of bone marrow
suppression e.g. inexplicable bruising or bleeding, infection.”

56
Q

Miss V has recently found out she is 6 weeks pregnant. She would like to purchase multivitamins as she has read that this is important to ensure a healthy pregnancy. Miss V has a previous medical history of asthma and sickle cell disease.
Which of the following would be the most appropriate advice for Miss V?
A. Miss V should avoid taking medications in pregnancy and instead eat a balanced, healthy diet with ginger if
required for nausea.
B. Miss V should purchase a multivitamin supplement over the counter which contains iron and folic acid.
C. Miss V should see her GP for a prescription for folic acid 5 mg.
D. Miss V should talk to her GP as she will need a blood test first to establish if she requires multivitamins.
E. Miss V should take folic acid 400 microgram each day to prevent neonatal defects.

A

C
see GP for folic acid 5 mg
Folic acid 5 mg daily should be prescribed during pregnancy to reduce the risk of neural tube defect and to
compensate for the increased demand for folate during pregnancy.
This strength is not available for purchase OTC and therefore a Rx is required.

57
Q

A patient has had a blood test and is found to have elevated thyroid function markers.
Which of the following medicines is least likely to cause hyperthyroidism?
A. Amiodarone
B. Calcium carbonate
C. Levothyroxine
D. Liothyronine
E. Lithium

A

B – calcium carbonate. All others can cause hyperthyroidism except calcium carbonate.
Calcium can reduce the absorption of levothyroxine, therefore this shouldn’t be taken at the same time as
levothyroxine.
Overtreatment with levothyroxine or liothyronine can cause hyperthyroidism.
Amiodarone and lithium can cause thyroid disorders.

58
Q

Pharmacists and pharmacy technicians must adhere to the nine GPhC Standards for Pharmacy Professionals.
One of these standards relates to safeguarding people, particularly children and vulnerable adults.
Which of the following is NOT one of the four principle categories of child abuse?
A. Emotional abuse
B. Modern slavery
C. Neglect
D. Physical abuse
E. Sexual abuse

A

B – modern slavery
All but modern slavery, which is one of the categories of abuse of adults.
WHO distinguishes four types of child maltreatment (emotional, sexual, physical, and neglect).

59
Q
Menopausal symptoms can be managed with hormone replacement therapy (HRT), however treatment benefits 
must be weighed against potential risks.
Which of the following conditions is least likely to develop as a result of using HRT?
A. Breast cancer 
B. Osteoporosis 
C. Stroke
D. Ovarian cancer
E. Venous thromboembolism
A

B (osteoporosis)
Risk of fragility fracture is DECREASED while taking HRT. This is beneficial, reducing risks of osteoporosis and fracture.

60
Q

A pharmacist is explaining inhaler technique to an adult patient newly prescribed a salbutamol pressurised
metered dose inhaler.
Which of the following statements regarding inhaler technique in this situation is most accurate?
A. Breathe in slowly and steadily through your mouth. Just after starting to breathe in, press the canister to release the dose. Continue to breathe in for 20 seconds, then hold your breath for a further 30 seconds, then breathe
out gently away from your inhaler.
B. Breathe in slowly and steadily through your mouth. Just after starting to breathe in, press the canister to release the dose. Continue to breathe in until your lungs feel full, then hold your breath for up to 10 seconds, or as long as is comfortable, then breathe out gently away from your inhaler.
C. Press the canister to release the dose, then breathe in slowly and steadily until your lungs feel full, then hold your breath for up to 10 seconds, then breathe out gently away from your inhaler.
D. Start breathing in quickly and deeply whilst simultaneously pressing the canister to release the dose. Continue breathing in quickly and deeply until your lungs feel full, then hold your breath for up to 10 seconds, then breathe out gently away from your inhaler.
E. Start breathing in quickly and deeply whilst simultaneously pressing the canister to release the dose. Continue breathing in quickly and deeply until your lungs feel full, then hold your breath for up to 10 seconds, then breathe out gently away from your inhaler.

A

B
The dose should only be released once the patient has their mouth around the mouth-piece and breathing in slow
and steady. Breath should be held for 10 seconds.
- 20 seconds is too long to breathe in for (try it!), the patient will be struggling for breath.
- If the dose is released before they start breathing in, the dose will not reach the lungs.
- Quick and deep breathing technique is used for dry powder inhalers, not pressurised MDIs.
GPhC Exam Framework: Identifying appropriate advice on the use of medicines (high), respiratory system (

61
Q

Following admission to the cardiology ward, a patient has their HAS-BLED score calculated. This score is taken into account before initiating a new medicine.
What purpose does the HAS-BLED tool serve?
A. Calculates the likelihood of a patient having a major cardiovascular event over the following ten years.
B. Determines a patient’s probability of having a deep vein thrombosis.
C. Estimates the risk of bleeding in patients with atrial fibrillation who are being offered anticoagulation.
D. Estimates the risk of bleeding in patients with heart failure who are taking immunosuppressants.
E. Estimates the risk of stroke for a patient with non-valvular atrial fibrillation.

A

C - Estimates the risk of bleeding in patients with atrial fibrillation who are being offered anticoagulation

A – describes QRISK3
B – describes the Wells score for DVT
D – incorrect
E – describes CHADSVASc

62
Q
Which of the following risk factors does a HAS-BLED score NOT take into account as part of the calculation?
A. Age
B. Alcohol use
C. Congestive heart failure
D. Hypertension
E. Liver disease
A

C, congestive heart failure, which is part of the CHADVASc stroke risk score.
See: https://www.nice.org.uk/guidance/qs93/chapter/Quality-statement-1-Anticoagulation-to-reduc

63
Q
An 82-year-old patient has been newly diagnosed with type 2 diabetes mellitus. Their past medical history includes atrial fibrillation, previous bladder cancer and hypertension. Recent blood results show an eGFR of 25 
mL/min.
Which of the following would be the most appropriate initial drug therapy for this patient?
A. Dapagliflozin
B. Glibenclamide
C. Metformin
D. Pioglitazone
E. Sitagliptin
A

E - sitagliptin
- Dapagliflozin unsuitable because it isn’t recommended in eGFR<60
- Glibenclamide unsuitable because there is a higher risk of hypoglycaemia in patients who are elderly or have
renal impairment, due to its longer action.
- Metformin is unsuitable due to this patient’s poor renal function. Higher risk of lactic acidosis in AKI.
- Pioglitazone is unsuitable due to history of bladder cancer being a contraindication.
Sitagliptin may be used for this patient, at a reduced dose for renal impairment.

64
Q

Neuroleptic malignant syndrome is a potentially life-threatening adverse effect related to the use of some antipsychotic and antiemetic drugs. It may affect up to 3% of patients taking antipsychotic agents and is
characterized by distinctive clinical signs and symptoms.
Which of the following symptoms is NOT part of the typical tetrad of symptoms associated with neuroleptic
malignant syndrome?
A Altered mental state
B Hyperthermia
C Muscle rigidity
D Red or purple rash
E Tachycardia

A

D – red or purple rash
The typical tetrad of symptoms of NMS are mental state change, muscular rigidity, hyperthermia and autonomic instability.

A red or purple rash is a sign of Stevens-Johnson Syndrome, associated with some antiepileptics among other medicines

65
Q

A ward pharmacist is monitoring the INR of patients prescribed warfarin. An INR which is within 0.5 units of the target value is considered to be satisfactory. For larger deviations, the pharmacist may make recommendations to medical staff on any necessary dosage adjustment.
Which of the following is the usual INR target value for a patient prescribed warfarin for atrial fibrillation?
A 1.0
B 2.0
C 2.5
D 3.5
E 3.75

A

C – 2.5
- 2.5 is the target value for many conditions including atrial fibrillation, treatment of DVT and PE, and
bioprosthetic heart values.
- 3.5 is used for recurrent DVT or PE in patients occurring in patients who were already on anticoagulation.
- A higher range (3.0 – 4.0 or 3.0 – 4.5) might be used for higher risk patients such as those with mechanical heart valves.
- 1.0 is the reference INR value, which would be expected for a normal person who isn’t on an anticoagulant

66
Q

A patient is admitted to hospital after developing slurred speech, a one-sided facial droop and confusion. They are diagnosed with an acute ischaemic stroke, and intracerebral haemorrhage is excluded by brain imaging. The patient has dysphagia and they are in atrial fibrillation.
What therapy should be initiated for this patient within 24 hours of presentation?
A Apixaban 5 mg plus atorvastatin 80mg orally
B Aspirin 300 mg orally
C Aspirin 300 mg plus atorvastatin 80mg orally
D Aspirin 300 mg rectally or by enteral tube
E Clopidogrel 300 mg rectally or by enteral tube

A

D - Aspirin 300 mg rectally or by enteral tube

  • Aspirin 300 mg orally if they do not have dysphagia or aspirin 300 mg rectally or by enteral tube if they do have dysphagia.
  • Ensure that people with disabling ischaemic stroke who are in atrial fibrillation are treated with aspirin 300 mg for the first 2 weeks before anticoagulation treatment is considered.
  • Immediate initiation of statin treatment is not recommended in people with acute stroke
  • Clopidogrel 75 mg is used for patients with aspirin hypersensitivity, or those intolerant of aspirin despite the addition of a proton pump inhibitor. A 300 mg loading dose is given for patients with STEMI or NSTEMI
67
Q

There are a number of lifestyle changes which can be made by people in mid-life to reduce the risk of developing dementia as they grow older.
Which of the following health promotion interventions is NOT related to dementia risk?
A. Achieve and/or maintain a healthy weight
B. Avoid air pollution
C. Be more physically active
D. Reduce alcohol consumption
E. Stop smoking

A

B – avoid air pollution
All except ‘avoid air pollution’ are listed in NICE Quality standard [QS184] on dementia, in quality statement 1:
Raising awareness – health promotion interventions.
Air pollution exposure is not linked to dementia but is a risk factor for developing asthma (so is obesity and smoking).

68
Q

You are running a weight loss clinic and one of your clients explains to you that whilst she knows she needs to start exercising, she just doesn’t have the motivation to start yet. She recently thought about buying an exercise bike to use at home, but then lost her job and can no longer afford it.
Which of the following stages of behaviour change is the client most likely at?
A. action
B. contemplation
C. maintenance
D. pre-contemplation stage
E. preparation

A

B - contemplation
In the contemplation stage, people acknowledge that they have a problem and begin to think seriously about solving it. They acknowledge the dangers and risks of their current behaviour and consider the pros and cons of changing.
While people in this stage may have vague plans to make changes, they are often not ready to take action yet. The person will still have reasons for continuing their behaviour. Many people remain in the contemplation stage for years.

69
Q

A 60 yo patient of black African family origin attends an annual health review at their GP surgery. They have type 2 diabetes treated with metformin, and osteoarthritis treated with co-codamol.
At their health review, they have high BP. This is confirmed with follow-up readings and it is decided they should start a new medicine to control their blood pressure.
Which class of antihypertensive drugs is the first-line choice for this patient according to NICE guidance?
A. Angiotensin II receptor antagonist
B. Beta blocker
C. Calcium channel blocker
D. Thiazide diuretic
E. Neprilysin inhibitor

A

A - Angiotensin II receptor antagonist
◇ ACEi or ARB is first choice step 1 treatment for people with type 2 diabetes regardless of age or family origin.
◇ If this patient didn’t have diabetes, choose a CCB. If they were black but under 55, choose a CCB.

■ NICE do not recommend initiating thiazide diuretics for HTN, but patients already on them and well-
controlled can continue Thiazide-like diuretics aren’t first line. Add a thiazide-like diuretic if step 1 treatment fails.
■ Beta blockers are only added if treatment resistant (i.e. ACEi/ARB + CCB + diuretic fails)
■ The neprilysin inhibitor sacubitril is only licensed for treatment of symptomatic chronic heart failure (in the
sacubitril/valsartan combination Entresto)

Step 1 treatment
1.4.30 》 Offer an ACEi or ARB to adults starting step 1 antihypertensive treatment who:
- have T2DB and are of any age or family origin (see also recommendation 1.4.29 for adults of black African or African–Caribbean family origin) or
- are aged under 55 but not of black African or African–Caribbean family origin.
1.4.31 》If an ACEi is not tolerated, e.g. cough SE, offer an ARB to treat hypertension.
1.4.32 》 Do not combine an ACEi + ARB to treat HTN.
1.4.33 》 Offer a CCB to adults starting step 1 antihypertensive treatment who:
- are aged 55 or over and do not have type 2 diabetes or
- are of black African or African–Caribbean family origin and do not have type 2 diabetes (of any age).
1.4.34 》If a CCB is not tolerated, e.g. oedema SE, offer a thiazide-like diuretic to treat HTN.
1.4.35 》If there is evidence of ❤F, offer a thiazide-like diuretic and follow NICE’s guideline on chronic heart
failure.
1.4.36 》If starting or changing diuretic treatment for hypertension, offer a thiazide-like diuretic, such as indapamide in preference to a conventional Thiazide diuretic such as bendroflumethiazide or hydrochlorothiazide.
1.4.37 》For adults with HTN already having treatment with bendroflumethiazide or hydrochlorothiazide, who
have stable, well-controlled blood pressure, continue with their current treatment

70
Q

A 65-year-old female was admitted to hospital for intense back and leg pain. Her right leg was weak and she reported a history of her legs buckling unexpectedly. She was also experiencing numbness in the groin area and in her legs. After undergoing an X-Ray and MRI, the medical team diagnosed cauda equina syndrome and the patient was transferred for emergency lumbar spine surgery.
Which of the following is also a symptom of cauda equina syndrome?
A. Fever
B. Loss of bladder control
C. Loss of consciousness
D. Sharp pain at the back of the head
E. Unexplained weight loss

A

B - Loss of bladder control
Symptoms of cauda equina syndrome:
• Severe low back pain.
• Motor weakness, sensory loss, or pain in one or commonly both legs.
• Saddle anaesthesia (unable to feel anything in the body areas that sit on a saddle)
• Bladder dysfunction (such as urinary retention or incontinence)
• Bowel incontinence.

71
Q

A patient visited their GP with symptoms which prompted an urgent referral under the suspected cancer pathway for an appointment within 2 weeks.
Which of the following is the most likely scenario which prompted the GPs decision?
A. A 24-year-old male with changes in their bowel habit for the last 7 days
B. A 36-year-old female with constipation for the last 3 days
C. A 48-year-old male complaining of diarrhoea and tiredness for the past 2 days
D. A 54-year-old male with unexplained rectal bleeding
E. A 61-year-old female with symptoms of indigestion and flatulence

A

D
CKS guidelines for suspected cancer pathway referral (for an appointment within 2 weeks) if:
• they are aged 40 and over with unexplained weight loss and abdominal pain or
• they are aged 50 and over with unexplained rectal bleeding or
• they are aged 60 and over with either iron-deficiency anaemia or changes in their bowel habit, or
• tests show occult blood in their faeces.
• Consider a suspected cancer pathway referral for colorectal cancer in adults with a rectal or abdominal mass.
• Consider a suspected cancer pathway referral for colorectal cancer in adults aged under 50 with rectal
bleeding and any of the following unexplained symptoms or findings:
- abdominal pain
- change in bowel habit
- weight loss
- iron-deficiency anaemia.

72
Q

You are reviewing new treatments for a condition and checking what the latest evidence base is for a particular drug. The drug you are interested in showed positive results in a cohort study and you are wondering whether or not this is a higher form of evidence compared that available for the other drugs.
Which of the following gives the most accurate description of a cohort study?
A. a study where a group of people are identified and followed over a period of time to see how their
exposures affect their outcomes
B. a study where people are randomly allocated to receive (or not receive) a particular intervention
C. a synthesis of the medical research on a subject, using thorough methods to search for and include all or as much as possible of the research on the topic.
D. a trial where neither the researchers nor the patients know what they are getting
E. an epidemiological study that describes characteristics of a population

A

A
A. a study where a group of people are identified and followed over a period of time to see how their exposures affect their outcomes. This type of study is normally used to look at the effect of suspected risk factors that can’t be controlled experimentally – for example, the effect of smoking on lung cancer.- Cohort
study
B. randomised controlled trial (RCT)
C. a synthesis of the medical research on a particular subject, using thorough methods to search for and include all or as much as possible of the research on the topic. Systematic review
D. a trial where neither the researchers nor the patients know what they are getting Double blind RCT
E. an epidemiological study that describes characteristics of a population Cross sectional study

73
Q

Vincristine, vinflunine and vinorelbine are all vinca alkaloid drugs used for a variety of cancers.
Which of the following describes the most appropriate method of administration for the above drugs?
A. intradermal
B. intramuscular
C. intrathecal
D. intravenous
E. subcutaneous

A

D – IntraVENOUS only.

Inadvertent intrathecal use can cause severe neurotoxicity, which is usually fatal.

74
Q

Which of the following doses of paracetamol, 120 mg/5 mL would be most appropriate for a 5-year-old boy?
A. 2.5 mL every 4–6 hours; maximum 4 doses per day
B. 5 mL every 4–6 hours; maximum 4 doses per day
C. 7.5 mL every 4–6 hours; maximum 4 doses per day
D. 10 mL every 4–6 hours; maximum 4 doses per day
E. 20 mL every 4–6 hours; maximum 4 doses per day

A

D

240 mg every 4–6 hours; maximum 4 doses per day.

75
Q

As part of information governance, pharmacy professionals must comply with the relevant data protection legislation.
Which of the following statements regarding data protection is CORRECT?
A. Data protection legislation applies to anonymised data as well as patient identifiable information
B. Hand written records are exempt from data protection requirements
C. Patient consent is required each time their data is accessed
D. Patients can consent by default to their data being collected, stored and processed
E. The Data Protection Officer must ensure non-compliance with data protection legislation is reported to the Information Commissioner’s officer within 72 hours

A

E - The Data Protection Officer must ensure non-compliance with data protection legislation is reported to the Information Commissioner’s officer within 72 hours.

Refer to the PJ article ‘How pharmacists can comply with GDPR’ from May 2018:
https://www.pharmaceutical-journal.com/cpd-and-learning/learning-article/how-pharmacists-can-comply-with-gdpr/20204872.article?firstPass=false

The key legislation is the Data Protection Act 2018 and the General Data Protection Regulation (GDPR).
A – anonymised data does not fall within this.
B – hand written records are still protected under data protection legislation.
C – consent would be required if it was being accessed for a different purpose to what was consented to originally e.g. you can use a patient’s phone number/email address to inform them their prescription is ready to collect if this is what they consented to, but you can’t email them with information about a new service unless they specifically consented to this first. We wouldn’t ask for consent to access a phone number every month before calling a patient with information regarding their Rx
D – Consent cannot be given by default or assumed, it must be gained.

Be aware of Data Protection Act 2018, and General Data Protection Regulation (GDPR), 2018

76
Q

A 5-year-old child has presented at the pharmacy with bullous impetigo. They are not systemically unwell or at high risk of complications. You suggest the patient makes a GP appointment.
Which of the following would be the most suitable initial treatment for the child?
A. A topical antibiotic such as fusidic acid 2% cream for 3 days
B. A topical antibiotic such as mupirocin 2% cream for 7 days
C. An oral antibiotic such as flucloxacillin 250 mg for 5 days
D. An oral antibiotic such as erythromycin 500 mg for 5 days
E. Hydrogen peroxide 1% cream for 5 days

A

C - An oral antibiotic such as flucloxacillin 250 mg for 5 days. Erythromycin is the other possibility, however it is only used in pregnancy for this condition. Clarithromycin would be used if patient is penicillin-alergic.
Impetigo is a highly contagious skin infection most commonly affecting young children. Sores (non-bullous impetigo) or blisters (bullous impetigo) usually develop on the face or hands.

Offer a short course of oral antibiotic for all people with:
● Bullous impetigo.
- Impetigo who are systemically unwell or at high risk of complications.
● For non-bullous impetigo, use topical hydrogen peroxide or topical antibiotic (fusidic acid or mupirocin).
- If widespread non-bullous impetigo, use a topical or oral antibiotic.

77
Q

You have been approached by a junior doctor who is reviewing a patient presenting with Clostridium difficile infection. The patient has recently finished a course of antibiotics but is unable to recall the name of them.
Which of the following is least likely to cause Clostridium difficile infection?
A. Aminoglycosides
B. Cephalosporins
C. Clindamycin
D. Co-amoxiclav
E. Quinolones

A

A - Aminoglycosides

Options B-E are all listed in the BNF as being able to cause C. diff infection.

78
Q

A 38-year-old woman presents at the pharmacy feeling sick and dizzy. You recognise her as a regular patient who recently brought in her new born baby. Upon further questioning, she explains that she feels confused and has severe muscle pain. You take her into the consultation room and notice her speech is slurred and she has cold and pale skin. She tells you that she has not urinated in the last 24 hours.
Which of the following conditions is she most likely suffering from?
A. diabetes
B. malaria
C. meningitis
D. sepsis
E. str

A
D
Patients should seek medical help urgently if they develop any of the following:
Slurred speech or confusion
Extreme shivering or muscle pain
Passing no urine (in a day)
Severe breathlessness
It feels like they’re going to die
Skin mottled, pale or discoloured

She has recently given birth, and if by C-section (i.e. an invasive procedure), this puts her at increased risk of sepsis.

79
Q

When preparing for surgery it is vital that the anaesthetist is aware of all medication that a patient is taking. The risk of losing disease control on stopping long-term medication before surgery is often greater than the risk posed by continuing it during surgery.
Which of the following would be least appropriate to stop prior to surgery?
A. Anticoagulants
B. Antiepileptics
C. Antiplatelets
D. Combined oral contraceptives
E. Hormone replacement therapy

A
B - Antiepileptics
- Patients taking antiplatelet medication or an oral anticoagulant present an increased risk for surgery.
- COCs and HRT have a VTE risk.
Drugs that should normally not be stopped before surgery include:
- Antiepileptics
- antiparkinsonian drugs
- Antipsychotics
- Anxiolytics
- Bronchodilators
- Some cardiovascular drugs
- glaucoma drugs, 
- immunosuppressants, 
- drugs of dependence, and 
- thyroid or antithyroid drugs. 
- Expert advice is required for patients receiving antivirals for HIV infection.
80
Q

You are preparing to run an “alcohol awareness” campaign to deliver in the community pharmacy.
According to NHS guidance, which of the following patients would it be most appropriate for you target to discuss reducing alcohol related health risks?
A. Mr E, a 76-year-old male, who drinks a total of 7 units at the weekend.
B. Mr K, a 24-year-old male who drinks a total of 12 units over 4 days.
C. Miss L, a 52-year-old female, who drinks 3 units on a Friday night.
D. Miss M, a 46-year-old female, who drinks 3 units a night, 4 days a week.
E. Mrs T, a 66-year-old female who drinks 4 units a day, every day.

A

E - 14 units is the maximum recommended weekly units for both men and women, therefore option E would be more than this.

To keep health risks from alcohol to a low level:
• men and women are advised not to drink more than 14 units a week on a regular basis
• spread your drinking over 3 or more days if you regularly drink as much as 14 units a week
• if you want to cut down, try to have several drink-free days each week

81
Q

An 83-yo woman with Parkinson’s disease is experiencing post-operative nausea and vomiting.
Which of the following antiemetics would be LEAST appropriate to recommend for this patient?
A. cyclizine IV
B. cyclizine IM
C. domperidone PO
D. metoclopramide PO
E. ondansetron IV

A

D - Metoclopramide

Metoclopramide can induce acute dystonic reactions involving facial and skeletal muscle spasms.

82
Q

You receive a prescription for cefalexin 125 mg/5 mL for an 8-year-old girl weighing 22 kg. The recommended dose is 12.5 mg/kg twice daily.
Which of the following is the correct volume per dose?
A. 5 mL
B. 8 mL
C. 11 mL
D. 12.5 mL
E. 25 mL

A

C
Dose is 12.5 mg x 22 kg = 275 mg twice daily
Product is 125 mg/5 mL, which is 25 mg/mL, therefore 275 mg in 11 mL.

83
Q

You are preparing 250 mL of a disinfectant solution for a patient. You explain to the patient that they must dilute the solution which you supply 1 in 10 before using it. This will then produce a solution with a concentration of one part per 5000, which is ready to apply to the patient’s skin.
What quantity of disinfectant, in grams, will be contained in the solution which you supply to the patient?
A. 0.01 g
B. 0.1 g
C. 0.5 g
D. 2.5 g
E. 5 g

A

C – 0.5 g
Final conc which the patient will use is 1g in 5000 mL, so the solution which the pharmacy supply is 10 x stronger than this i.e. 1g in 500 mL.
There is therefore 0.5 g in the 250 mL which we supply the patient

84
Q

Agranulocytosis is a rare condition in which bone marrow doesn’t make enough of a certain type of white cell, most often neutrophils.
Which of the following drugs should NOT be used in combination with methotrexate due to the increased risk of agranulocytosis?
A. Aspirin
B. Clomipramine
C. Olanzapine
D. Pioglitazone
E. Tramadol

A

C – Olanzapine

Rationale: both drugs have high risk of agranulocytosis

85
Q

Every time a pharmacy professional renews their registration with the GPhC, they must submit records to show how they have carried out and recorded revalidation activities.Which of the following statements regarding revalidation is CORRECT?
A. Each year, and by the time a pharmacy professional renews their registration, the GPhC will expect them to carry out, record and submit nine CPD entries, at least two of which must be planned learning activities.
B. Each year, and by the time a pharmacy professional renews their registration, the GPhC will expect them to carry out, record and submit two peer discussions and two reflective accounts.
C. Planned learning is a learning and development activity that encourages you to engage with others in your reflection on learning and practice.
D. The purpose of the reflective account is to encourage pharmacy professionals to think about how they meet
the GPhC standards for pharmacy professionals in the work which they do.
E. Unplanned learning is when you decide to develop your knowledge and/or skills in advance of carrying out
the learning activity.

A

D
- Option A is incorrect as the GPhC will expect them to submit four CPD entries, at least two of which must be
planned learning activities.
- Option B – only one peer discussion and one reflective account required
- Options C describes peer discussion
- Option E describes planned learning

86
Q

A 2-year-old child has chicken pox and requires treatment over-the-counter for the itch. They have no other medical conditions and do not take any other medication.
Which of the following would be the most appropriate to recommend?
A. cetirizine 5 mg twice daily
B. chlorphenamine 1 mg every 4-6 hours
C. hydroxyzine 50 mg twice daily
D. loratadine 1 mg once daily
E. promethazine 25 mg twice daily

A
B
A is the dose for aged 6+
C is a POM and an overdose
D is an incorrect dose (should be 5 mg or 10 mg depending on the child’s weight )
E is an overdose
87
Q

A patient presents with symptoms of sneezing, nasal itching, rhinorrhoea and nasal congestion. You diagnose allergic rhinitis and explain to the patient which products are available over-the-counter. The patient decides to buy a Clarinaze® Allergy Control Nasal Spray, containing mometasone 0.05%.
Which of the following statements regarding the use of the Clarinaze® Allergy Control Nasal Spray is
INCORRECT?
A. Before first administration the spray should be primed, this is achieved by actuating the pump ten times until a fine mist is obtained; re-prime with two sprays if the pump has not been used for 14 days
B. Clarinaze® Allergy Control Nasal Spray may be supplied to adults over 12 years of age.
C. Side effects include epistaxis, headache, nasal discomfort and irritated throat
D. The active ingredient, mometasone, is a corticosteroid.
E. The recommended dose is 100 mcg into each nostril once a day until symptoms are controlled, then 50 mcginto each nostril daily.

A

B – the licensed minimum age is 18 years. All other statements are true.

88
Q

Clinical audit is vital for patient care and safety, as well as for the development and improvement of
professional practice.
Which of the following statements regarding clinical audit is INCORRECT?
A. Part of the process of clinical audit is to compare your service to existing standards or criteria.
B. Patient consent will not be required if you are collecting data which you already provide as part of your care to the patient, and which does not identify the patient.
C. The aim of a clinical audit is to determine whether a current service or procedure reaches a specified
standard, to use that information to inform improvements in care, and then evaluate those changes by re-auditing.
D. The re-audit phase of the audit cycle involve collecting a second set of data and analysing the results to determine whether the changes made in the initial audit have been effective.
E. You must receive ethical approval before conducting a clinical audit.

A

E - You do not need ethics approval to conduct a clinical audit.

89
Q

Mr R has been prescribed colchicine for an acute attack of gout.
Which of the following statements regarding gout and the use of colchicine is INCORRECT?
A. Certain drugs can increase the risk of developing gout by raising plasma urate levels, such as ACE inhibitors, beta-blockers and diuretics.
B. Colchicine should be avoided in people with an eGFR less than 10, women who are pregnant or breastfeeding, or those with severe hepatic impairment.
C. Mr R should have 500 micrograms of colchicine, two to four times a day, until pain relief is achieved, or diarrhoea or vomiting occurs.
D. Mr R should be advised to not exceed a total dose of 6 mg of colchicine and to not repeat treatment within three days.
E. Purine-rich foods, such as green leafy vegetables, increase the risk of gout, as higher purine intake is associated with uric acid levels above 150 micromol/L

A

E – purine-rich foods, particularly red meat and seafood, increase the risk of gout as higher purine intake is associated with higher uric acid levels.
For maintenance treatment, the aim is for serum uric acid levels below 300 micromol/L.

90
Q

A patient, aged 31, is attending a community pharmacy-led travel clinic. The patient is planning a trip around South East Asia for 12 months and asks your advice regarding vaccinations.
Which of the following statements regarding the use of hepatitis B vaccine is INCORRECT?
A. An accelerated schedule is possible, which for adults is a 20 mcg dose at zero, one and two months, with a fourth dose at 12 months for those in certain risk categories.
B. Symptoms of hepatitis B may include jaundice, loss of appetite, fever and abdominal pain.
C. The deltoid muscle is the preferred site for the intramuscular injection.
D. The standard immunisation schedule for adults is 20 mcg at zero, one and six months.
E. Those at high-risk of hepatitis B include travellers to endemic areas where frequent or prolonged exposure to poor sanitation and poor food hygiene is likely

A

E. Those at high-risk of hepatitis B include travellers to endemic areas where frequent or prolonged exposure
to poor sanitation and poor food hygiene is likely. – this is the case for typhoid

E, which currently describes the case for typhoid.
Behaviours and activities which increase the risk of Hepatitis B include:
• unprotected sex.
• exposure to blood or blood products through occupation, such as healthcare work.
• exposure to contaminated needles through injecting drug use, or as a result of accessing medical or dental
care, because of needing emergency treatment or those travelling specifically for medical treatment.
• participation in contact sports.
• adoption of children from risk countries.
• long stay travel.

91
Q
A 42-year-old woman asks for your advice on the management of rosacea. She tells you that she has read conflicting reports of the most common factors which may trigger or worsen her symptoms.
Based on current evidence, which of the following factors is LEAST likely to trigger or worsen rosacea 
symptoms?
A. Emotional stress
B. Exercise 
C. Ice-cold drinks
D. Smoking 
E. Spicy foods
A
C – hot drinks are a trigger, not cold drinks
Factors include:
• Increasing age.
• Photosensitive skin types.
• Ultraviolet radiation exposure.
• Smoking.
• Heat or cold ambient temperature.
• Spicy foods and hot drinks.
• Alcohol.
• Emotional stress and exercise.
• Drugs such as calcium-channel blockers (may worsen vasodilatation and flushing) and topical corticosteroids.
92
Q

Topical corticosteroids are classified according to their potency, being either mild, moderate, potent or very potent.
Which of the following topical corticosteroids is classified as moderate?
A. Betnovate® (betamethasone valerate)
B. Dermovate® (clobetasol propionate)
C. Eumovate® (clobetasone butyrate)
D. Hc45® (hydrocortisone)
E. Nerisone Forte® (diflucortolone valerate)

A

C – Eumovate®
See BNF chapter 13, topical corticosteroids, for more information.
A. Betnovate® (Betamethasone Valerate) - potent
B. Dermovate® (Clobetasol propionate) – very potent
C. Eumovate® (clobetasone butyrate) - moderate
D. Hc45® (hydrocortisone) - mild
E. Nerisone Forte® – very potent

93
Q

PPI inhibit gastric acid secretion by blocking the ‘proton pump’ of the gastric parietal cell. Whilst effective and commonly used medicines, they have several cautions and monitoring requirements associated with them.
Which of the following cautions is NOT associated with proton pump inhibitors?
A. Measurement of serum-magnesium concentrations should be considered before and during prolonged treatment with a proton pump inhibitor.
B. Patients at risk of osteoporosis should maintain an adequate intake of calcium and vitamin D, due to the increased risk of fractures with proton pump inhibitors.
C. Particular care is required for patients presenting with any ‘alarm features’ of prostate cancer, due to the ability of proton pump inhibitors to mask the symptoms of prostate cancer in adults.
D. Proton pump inhibitors may increase the risk of gastro-intestinal infections, including Clostridioides difficile infection.
E. Proton pump inhibitors should always be prescribed for appropriate indications at the lowest effective dose for the shortest period of time.

A

C
Particular care is required for patients presenting with any ‘alarm features’ of gastric cancer, due to the ability of proton pump inhibitors to mask the symptoms of gastric cancer in adults

94
Q

It is normal practice when dispensing atenolol to add which of the following cautionary labels to the container?

A. Dissolve or mix with water before taking
B. This medicine may colour your urine. This is harmless
C. Warning: Do not drink alcohol
D. Warning: Do not stop taking this medicine unless your doctor tells you to stop
E. Warning: May cause drowsiness. If affected do not drive or operate machinery

A

D - Sudden cessation of beta-blockers may cause a rebound worsening of myocardial ischaemia & may
cause exacerbation of angina. See BNF chapter 2, section 4.1, Beta-adrenoceptor blocking drugs and recommended wording of cautionary and advisory labels at the very back of the BNF.

95
Q

When supplying inhaled corticosteroids, there are several important counselling points which should be
discussed with the patient and/or carer.
Which of the following statements regarding the use of inhaled corticosteroids is CORRECT?
A. A steroid card is only required to be supplied to patients taking oral corticosteroids and not those who are only on inhaled corticosteroids.
B. If oral candidiasis develops, treatment should be stopped while an anti-fungal suspension or oral gel is being used as treatment.
C. Qvar® and Clenil® Modulite are interchangeable, as both are beclometasone dipropionate CFC-free pMDIs.
D. The height and weight of children receiving prolonged treatment with inhaled corticosteroids should be monitored annually.
E. The risk of oral candidiasis is increased by using a spacer device with a corticosteroid inhaler.

A

D – referral to paediatrician should be considered if growth is slowed

96
Q

A 13-year-old child has been prescribed carbimazole. You have arranged to speak with the child’s parents regarding the use of the new medicine.
Which of the following symptoms is LEAST likely to be a sign of agranulocytosis or neutropenia?
A. bruising
B. diarrhoea
C. fever
D. malaise
E. sore throat

A

B – The other options are all listed in the ‘Patient and Carer Advice’ section of the drug monograph.

97
Q

Pharmacists must be first aid trained to ensure they are able to manage medical emergencies in the community.
In a community setting with a conscious adult patient, which of the following would be the most appropriate first-line emergency medical treatment for a patient with diabetes suffering from hypoglycaemia?
A. A 500 mL glass of pure fruit juice
B. Approximately 2.5 g of glucose
C. Approximately 20 g of sucrose
D. Glucose 10% intravenous infusion
E. Glucagon injection 1 mg

A

C
A. 150–200 mL pure fruit juice would be appropriate - 500 mL too much
B. Approximately 2.5 g of glucose – approx. 15-20 g is recommended of glucose or sucrose
C. Approximately 20 g of sucrose – CORRECT
D. Not first line and wouldn’t be available immediately - Glucose 10% intravenous infusion
E. Glucagon injection 1 mg – only if hypoglycaemia unresponsive or if oral route cannot be used

98
Q

Miss Y has come to collect her repeat prescription for Ovranette® tablets (ethinylestradiol 30micrograms and levonorgestrel 150 micrograms). When handing out her medication, Miss Y explains that she recently missed a pill and is not sure whether she still has contraceptive protection.
What of the following statements regarding missed pills for combined hormonal contraceptives is CORRECT?
A. A missed pill is one that is 12 or more hours late
B. A missed pill is one that is 24 or more hours late
C. A missed pill is one that is 48 or more hours late
D. A missed pill is one that is 72 or more hours late
E. A missed pill is one that is 120 or more hours late

A

B – 24 hours or more late
A missed pill is one that is 24 or more hours late. If a woman misses only one pill, she should take an active pill as soon as she remembers and then resume normal pill-taking. No additional precautions are necessary.

99
Q

Mrs T has been admitted to A&E having vomited fresh blood. Her current medication list is shown below:

  • Atorvastatin 40 mg od
  • Metformin 500 mg tds
  • Naproxen 500 mg bd
  • Paracetamol 500 mg prn
  • Valsartan 160 mg od

Which of Mrs T’s current medicines is most likely to be causing her symptoms?

A. Atorvastatin 
B. Metformin 
C. Naproxen 
D. Paracetamol 
E. Valsartan
A

C naproxen

100
Q

A Pharmacy Technician asks for your advice regarding a recent increase in dispensing errors across the pharmacy team. She would like to know how she can train her colleagues to minimise the risk of making future errors.
Which of the following statements would be the least appropriate advice to give?

A. Arrange a meeting with all staff to encourage reporting of errors to help identify any changes required in the dispensing process or layout of the dispensary.
B. Create a near miss log which tracks all the errors made and then analyse the near misses to look for patterns which may suggest a reason for the increase in errors
C. Create a short mental break once the medicines are assembled and before completing the accuracy check.
D. Use the medicine labels to select products from the dispensary shelves, rather than dispensing from the prescription.
E. When dispensing balances of prescription items which are owed, refer to the patient medication record for what was previously dispensed.

A

D

101
Q

The Home Office has advised that all controlled drugs (CDs) in Schedules 2, 3 and 4 (part 1) should be denatured and, therefore, rendered irretrievable before disposal.
Which of the following statements regarding the denaturing of CDs is INCORRECT?
A. Aerosol formulations should be expelled into water and the resulting liquid disposed of in an appropriately-sized CD denaturing kit.
B. An authorised witness is not required to denature a patient-returned Schedule 3 CD
C. An authorised witness is not required to denature an expired Schedule 2 stock item.
D. It is advisable to grind or crush solid dose formulations before adding them to a CD denaturing kit to ensure that whole tablets or capsules are not retrievable.
E. Patches can be destroyed by removing the backing and folding the patch over on itself.

A

Answer: C

An authorised witness is required to denature any expired Schedule 2 stock.

102
Q

A patient has received their first prescription for glyceryl trinitrate 500 microgram tablets.
Which of the following statements describing the shelf life and storage requirements is CORRECT?
A. The glass container has cotton wool wadding inside to prevent the tablets from breaking.
B. The tablets are flammable and must be kept away from fire or flames.
C. The tablets can be removed from their glass container and placed in a dossette box if required.
D. The tablets should be discarded 8 weeks after first opening.
E. The tablets should be discarded 12 weeks after first opening.

A

D
If you do not use the tablets within 8 weeks of first opening the bottle, obtain a fresh supply and return the old tablets to the pharmacist for disposal.
The tablets should not be removed from their amber glass container, which has a screw cap lined with aluminium
foil.
Glyceryl trinitrate tablets should not contain cotton wool wadding.
The s/l spray is flammable and should be kept away from flames.

103
Q

Mrs V is experiencing nightmares which are disturbing her sleep. Her GP contacts you to ask whether one of her medicines could be causing the nightmares.
Which of the following is least likely to cause nightmares as a side effect?
A. Labetalol
B. Oxprenolol
C. Propranolol
D. Sotalol
E. Timolol

A

D – sotalol is water soluble and less likely to permeate the blood brain barrier
Water soluble – atenolol, sotalol, nadolol – less likely to enter the brain, therefore less likely to cause sleep
disturbance and nightmares
Lipid soluble – labetalol, oxprenolol, propranolol, timolol

104
Q

An elderly patient is suffering with constipation and requires treatment with a suitable laxative. She is not drinking much liquid at the moment and would like something to take at night before bed.
Which of the following laxatives would be least appropriate?
A. Bisacodyl tablets
B. Ispaghula husk granules
C. Lactulose oral solution
D. Senna tablets
E. Sodium picosulphate oral solution

A

B – Fluid intake must be maintained with bulk-forming laxatives to avoid intestinal obstruction. They should
not be taken immediately before going to bed.

105
Q

Mrs C is 54 years old and has been commenced on sulfasalazine 1.5g four times a day by her GP two weeks ago, alongside prednisolone 20mg daily for treatment of inflammatory bowel disease. She enters your pharmacy complaining of having a sore throat, fever and has a few nosebleeds in the last 24 hours. She wonders if her new tablet is to blame.
Which is the most appropriate advice for this patient?
A. The symptoms described are not known to be caused by sulfasalazine
B. She should see her GP as the dose of sulfasalazine may need to be increased
C. She should see her GP as the dose of sulfasalazine may need to be reduced
D. She is experiencing a side-effect of sulfasalazine, and whilst safe to continue taking it, she may wish to see
her GP for an alternative
E. Stop taking sulfasalazine straight away and see her GP as soon as possible

A

E - Blood disorders can occur with aminosalicylates, the drug should be stopped immediately if there is a
suspicion of a blood dyscrasia. (section 1.2

106
Q

A 37-year-old woman presents at the pharmacy with a red eye. She explains that her eye has been red and irritated for the past day or so. She asks to buy some over-the-counter drops for it.
Which of the following symptoms would require an urgent referral to the nearest eye hospital?
A. Excessive watering of the eye.
B. Photophobia with pain originating from inside the eye.
C. Slightly blurred vision following a long period of computer working.
D. Stiff neck and tense shoulders alongside red eyes.
E. Yellow, sticky discharge and discomfort in the eye.

A
B
Alarm symptoms related to the eye include:
- Photophobia
- Pain inside the eye
- Loss of vision
107
Q

Mr C has been diagnosed with H. Pylori infection. He also takes Ciclosporin 100mg BD for a kidney transplant which he received 2 years ago.
Which of the following H. Pylori treatments would be the best option for him?

Esomeprazole 20mg BD, Amoxicillin 1g BD, Clarithromycin 500g BD for 7 days
Lansoprazole 30mg BD, Amoxicillin 1g BD, Metronidazole 400mg BD for 7 days
Omeprazole 20mg BD, Clarithromycin 250mg BD, Metronidazole 400mg TDS for 7 days
Lansoprazole 30mg BD, Clarithromycin 250mg BD, Metronidazole 400mg BD for 7 days
Omeprazole 20mg BD, Amoxicillin 1g BD, Clarithromycin 500mg BD for 7 days

A

Lansoprazole 30mg BD, Amoxicillin 1g BD, Metronidazole 400mg BD for 7 days – Clarithromycin increases the concentration of Ciclosporin. SEVERE INTERACTION

108
Q

A 34-year-old woman who is 16 weeks pregnant and suffering from constipation, despite following the suggested lifestyle and dietary changes. Her GP suggests the next recommended course of action.
Which of the following is the GP most likely to recommend?

Senna tablets
Macrogols
Diarolyte oral sachets
Liquid paraffin
Isphagula husk granules
A

Isphagula husk granules

If dietary and lifestyle changes fail to control constipation in pregnancy, fibre supplements in the form of BRAN OR WHEAT are likely to help women experiencing constipation in pregnancy, and raise no serious concerns about side-effects to the mother or foetus. Bulk-forming laxative is the first choice during pregnancy if fibre supplements fail. An an osmotic laxative, such as LACTULOSE, can also be used.

109
Q

While conducting an MUR for Mr P, a 70-year-old man wo suffers from hypothyroidism, bronchitis and bipolar disorder, you note that Mr P is also taking Lansoprazole 30mg daily. He is not experiencing symptoms anymore. You recommend that the GP review the medication as there is a risk of complications that may arise from long term use of PPIs.

Which of the following is NOT a risk factor of long term use of PPIs?

Clostridium Difficile infection
Fractures
Hypomagnesaemia
Constipation
Gastric cancer
A

Constipation

PPIs: can increase risk of fractures (high doses for over a year in elderly & adults); may increase risk of GI infections (C Diff); may mask the symptoms of gastric cancer; patients at risk of osteoporosis

110
Q

Miss K is 21 years old and is complaining of a flare of her IBS, previously diagnosed by their GP. She has symptoms of abdominal pain and cramping after meals. She has had the symptoms before but cannot recall what she took to help. She is otherwise healthy with no known drug allergies. Normally, she manages her diet well, avoiding triggers. She is not pregnant.
Which agent would be the most appropriate choice to control her symptoms?

A. Atropine
B. Domperidone
C. Hyoscine Butylbromide
D. Metoclopramide
E. Promethazine
A

Hyoscine Butylbromide – An antispasmodic used to relieve cramping in IBS

111
Q

Mr S has been diagnosed with a mild Clostridium difficile infection for the first time on one of your wards.
Which of the following of Mr S’ medicines would you withhold and review in relation to the C. Diff infection?

Calcichew D3 forte, 1 tablet BD
Losartan 50mg OM
Lansoprazole 30mg OM 
Paracetamol 500mg QDS
Prednisolone 10mg OM
A

Lansoprazole 30mg OM – PPIs can increase the risk of GI infections including C. Diff infection

112
Q

Mrs P is one of the patients on your oncology ward and has recently had her analgesia increased to Oxycontin (oxycodone MR) 20mg BD alongside OxyNorm (oxycodone) 5mg PRN. A few days later, upon inspecting her stool chart, you notice that she has not opened her bowels for the past 2 days. You discuss with the medical team and agree she needs a laxative.

Which of the following options would be most appropriate?

Lactulose 10mL BD PRN
Senna 15mg ON PRN
Lactulose 10mL BD and Senna 7.5mg-15mg ON PRN
Lactulose 10mL BD and Movicol (Macrogol powder) 1 sachet ON PRN
Mrs P should be referred for urgent bowel evacuation

A

Lactulose 10mL BD and Senna 7.5mg-15mg ON PRN

Constipation in palliative care patients who are receiving opioids should be prevented with the regular administration of laxatives; A FAECAL SOFTENER WITH A PERISTALTIC STIMULANT (e.g. co-danthromer) or LACTULOSE SOLUTION WITH A SENNA PREPARATION should be used. METHYLNALTREXONE BROMIDE is licensed for the treatment of opioid induced constipation.

113
Q

Mr T has been diagnosed with a blocked bile duct and has been prescribed the drug colestyramine in an attempt to control the itching he is experiencing. He enquires about the potential side effects of the drug.
Which one of the following adverse effects is not normally associated with colestyramine?

Constipation
Increased tendency to bleed
Myalgia
Nausea
Reduced absorption of Vitamins A, D, E, and K
A

Myalgia

Other s/e include: Diarrhoea, GI discomfort, hypertriglyceridaemie (aggravation), hypoprothrombinaemia associated with Vit K deficiency, vomiting

114
Q

A patient has been taking clindamycin for the past three days and has developed nausea and diarrhoea. They ask for your advice about what to do. They have been taking Clindamycin as prescribed: 450mg QDS. They still have four more days of treatment to go.
What is the most appropriate advice to give the patient?
A They should continue with the treatment as usua
B They should go to A&E immediate
C They should see their GP to get an alternative medication
D They should stop taking the Clindamycin immediately and go and get an urgent appointment with their doctor
E They should take an over the counter medicine for the nausea and diarrhoea

A

D

With systemic use, clindamycin has been associated with antibiotic-associated colitis, which may be fatal. Patients should, therefore, discontinue treatment immediately if diarrhoea develops.

115
Q

Naeem was diagnosed with type 2 diabetes several years ago. He has struggled to control the condition and has suffered from fluctuating blood sugars and periods of low mood. He’s come to talk to you today because he has been feeling odd recently, with episodes of dizziness and light-headedness, often when he first stands up. He has mild diarrhoea and some difficulty passing water.

His current prescription consists of:

amlodipine 5 mg daily
atorvastatin 40 mg daily
duloxetine 60 mg daily
glimepiride 4 mg daily
indapamide 2.5 mg daily
metformin 500 mg three times a day

What is the most likely explanation for this combination of symptoms?

A. Autonomic neuropathy
B. Dehydration
C. Hypoglycaemia
D. Lactic acidosis
E. Postural hypertension
A

C. Hypoglycaemia; glibenclamide and metformin

**CPPE

116
Q

Sunita was diagnosed with type 2 DB around 3 years ago and she had been well-controlled within her target (48 mmol/mol) using a combination of lifestyle modification and metformin MR 2 g once daily. However, her HbA1c has risen to 51 mmol/mol since her last routine review.

Having confirmed Sunita’s adherence to metformin, which ONE of the following is the most appropriate initial course of action in this case?
A. Add DPP4 inhibitor
B. Add sulphonylurea
C. Change Metformin MR dose to 1g twice daily
D. Check whether any lifestyle modifications can be made
E. Consider adjusting HbA1c target

A

D. Lifestyle modifications? WRONG

**CPPE

Note: Consider relaxing the target HbA1c level on a case-by-case basis, with particular consideration for people who are older, frail, or where tight blood-glucose control is not appropriate or poses a high risk of the consequences of hypoglycaemia.

117
Q

Self-monitoring of blood glucose is not considered a routine part of management in people with type 2 DB, but it is still important in some circumstances. Carl is considering his options for 1st intensification since metformin is no longer controlling his HbA1c to target. He isn’t keen on the idea of self-monitoring.

Which ONE of the following oral blood glucose lowering medicines would be most likely to require self-monitoring of blood glucose?
A. Gliclazide
B. Canagliflozin
C. Linagliptin
D. Liraglutide
E. Pioglitazone
A

gliclazide??

** CPPE

118
Q

Sylwia started taking metformin for type 2 diabetes about a year ago. She’s just got engaged and is starting to think about having a family. She asks what HbA1c target she needs to aim for.

Which ONE of the following HbA1c targets does the NICE guideline recommend for preconception care?
A. 48 mmol/mol
B. 53 mmol/mol
C. 58 mmol/mol
D. 68 mmol/mol
E. 86 mmol/mol
A

A. 48 mmol/mol
** CPPE

Type 2 DB should initially be treated with a single oral antidiabetic drug. A target HbA1c of 48 mmol/mol (6.5%) is generally recommended when managed by diet and lifestyle alone or when combined with a single antidiabetic drug not associated with hypoglycaemia (such as metformin hydrochloride). Adults prescribed a single drug associated with hypoglycaemia (such as a sulphonylurea), or two or more antidiabetic drugs in combination, should usually aim for an HbA1c concentration of 53 mmol/mol (7.0%).

119
Q
Margaret is a 63-yo retired secretary who has recently been diagnosed with type 2 DB. Lifestyle modification has made a modest difference to her HbA1c, but isn’t sufficient. Her current prescription consists of:
bisoprolol 2.5 mg daily
ferrous sulphate 200 mg twice daily
furosemide 40 mg daily
ramipril 10 mg daily

Her baseline HbA1c is 59 mmol/mol and her eGFR is 28 ml/min/1.73m2. Full blood count, liver function tests and cholesterol are unremarkable.

Which ONE of the following would be the most appropriate medicine for Margaret to take according to current NICE guidance?

A. DPP4 inhibitor
B.  Sulphonylurea
C. Acarbose
D. metformin
E. Pioglitazone
A

D. metformin? WRONG
=> Manufacturer advises avoid if eGFR is less than 30 mL/minute/1.73 m2.

Correct answer should then be DPP4 inhibitor??
** CPPE

120
Q

John has recently been diagnosed with type 2 diabetes and he has been offered a structured education programme, but he is not sure whether it is worth attending.

Which ONE of the following outcomes has been demonstrated in newly diagnosed adults with type 2 diabetes who undertook the DESMOND programme?

A. Changes to illness beliefs
B. Delayed need for insulin
C. Improved glycaemic control
D. Reduced complications at 10 years
E. Reduced need for oral blood glucose lowering medicines
A

A. Changes to illness beliefs
** CPPE
CORRECT

121
Q

Marie has come to see you for a routine review of her type 2 diabetes. She has been taking metformin 500 mg tds for the last 3 months and her HbA1c has fallen to 45 mmol/mol (6.2 percent).

Which ONE of the following is the most appropriate option to suggest to Marie?

A. Add DPP4 inhibitor
B. Add SGLT2 inhibitor
C. Change metformin to DPP4 inhibitor
D. Continue with current regimen
E. Increase Metformin dose by 500mg per day
A

D. continue with current regimen
** CPPE

(Believe this is correct as target is 48 mmol/mol and she is below this now which is good)

122
Q

Lukas has been managing his type 2 DB for over 10 years and his HbA1c has been slowly rising recently. He was unable to tolerate metformin and is aware that his current regimen of gliclazide 160 mg each morning is not working well enough. He hasn’t been as active as usual recently and he’s concerned about weight gain. In fact he’s quite keen to try to lose some weight.

Which ONE of the following options would be most appropriate for Lukas?
A. Add pioglitazone 15mg once daily
B. Change gliclazide to pioglitazone 15mg once daily
C. Change gliclazide to sitagliptin 100mg daily
D. Increase gliclazide to 240mg once daily

A

C. Change gliclazide to sitagliptin 100mg daily
** CPPE

Pioglitazone => weight gain
Gliclazide => weight gain

123
Q

Freddie was diagnosed with DB over 25 years ago and moved into a nursing care home following a fall a few days ago. His current repeat prescription comprises:

Adcal D3 chewable tablets, 1 twice daily
atorvastatin 10 mg tablets, 1 daily
glibenclamide 5 mg tablets, 2 daily
metformin 500 mg tablets, 1 with breakfast and lunch, and 2 with the evening meal
paracetamol 500 mg tablets, 1 or 2 every 4 to 6 hours when needed for pain relief
ramipril 5 mg capsules, 1 daily

His nurses have noticed that his blood glucose has been falling since he arrived.

Which ONE of the following is the most appropriate action to address Freddie’s hypoglycaemia?

A. Reduce dose of glibenclamide
B. Reduce the dose of metformin
C. Stop Glibenclamide
D. Stop Metformin
E. Stop Glibenclamide and Metformin
A

A. Reduce dose of glibenclamide??

** CPPE

124
Q

Miriam has just called to find out the results of her HbA1c and blood pressure monitoring. Her HbA1c is within her target. Her blood pressure is 145/85 mmHg.

What does the NICE guideline recommend for an adult with type 2 diabetes with blood pressure 145/85 mmHg?

A. Take another measurement at next consultation
B. 
C. 
D. 
E. Offer ACE inhibitor right away
A

I think A. ?

NICE: If blood pressure measured in the clinic is 140/90 mmHg or higher:
Take a second measurement during the consultation.
If the second measurement is substantially different from the first, take a third measurement.

Record the lower of the last 2 measurements as the clinic blood pressure.
**CPPE